Site MapHelpFeedbackQuiz 1
Quiz 1
(See related pages)

1
Mainstream economists claim the increased instability of the economy over the last 40 years supports their "hands-off" policy prescription.
A)True
B)False
2
"A recession became the Great Depression because the Fed allowed the money supply to fall by 35% during the 1930s." This statement is most closely associated with which school of thought?
A)Keynesianism
B)Real business cycle theory
C)Rational expectations theory
D)Monetarism
3
The inability of outsiders to underbid insiders for jobs suggests that:
A)the economy will self-correct slowly, if at all
B)the natural rate of unemployment will fall during a recession
C)the Phillips curve will be vertical
D)work-place morale can be improved with "two-tier" wage systems
4
The new classical view suggests that:
A)wages are inflexible downward
B)coordination failures lead to persistent unemployment
C)the long-run aggregate supply curve is vertical
D)the long-run Phillips curve is downward sloping
5
Real business cycle theorists conclude that the primary reason for cyclical changes in real GDP is inappropriate fiscal and monetary policy.
A)True
B)False
6
Suppose nominal GDP is $5000 billion, real GDP is $4000 billion, and the money supply is $1000 billion. In this hypothetical economy, the velocity of money is:
A)5
B)4
C)4.5
D)$1000
7
The Keynesian assumptions regarding wages and prices suggest that a decrease in aggregate demand will:
A)leave output and the price level unchanged
B)reduce real output and the price level
C)reduce real output but not the price level
D)reduce the price level but not real output
8
Answer the next question on the basis of the following information for a hypothetical economy. All values are in nominal terms.
M = $400
V = 4
C = $1280
Ig = $160
G = $80
Xn = $80

Refer to the information. If the price level is 2, Q (real GDP) is:
A)$1600
B)$400
C)$800
D)$2000
9
Suppose the money supply is $2000 billion and the velocity of money is 5. If the price level is 4, nominal GDP is:
A)$2000 billion
B)$2500 billion
C)$10,000 billion
D)$40,000 billion
10
According to the monetarist view:
A)changes in velocity are small and predictable
B)velocity is inversely proportional to nominal GDP
C)the equation of exchange holds true only at full-employment GDP
D)adverse supply shocks are the primary cause of monetary instability







McConnell, Macro 17e OLCOnline Learning Center

Home > Chapter 17 > Quiz 1